Are unique prime ideal factorization domains locally noetherian?
$begingroup$
In this question I asked: "Are unique prime ideal factorization domains noetherian?".
In this answer Badam Baplan pointed out that locally noetherian domains are unique prime ideal factorization domains (UPIFD), that certain non-noetherian domains are locally noetherian (the first example seems to have been given by N. Nakano in 1953), and thus that these non-noetherian domains are UPIFD.
This prompts the present question:
Are unique prime ideal factorization domains locally noetherian?
Edit. For the sake of completeness recall that a UPIFD is a domain satisfying the following condition:
If $mathfrak p_1,dots,mathfrak p_k$ are distinct nonzero prime ideals of $A$, and if $m$ and $n$ are distinct elements of $mathbb N^k$, then we have
$$
mathfrak p_1^{m_1}cdotsmathfrak p_k^{m_k}nemathfrak p_1^{n_1}cdotsmathfrak p_k^{n_k}.
$$
commutative-algebra maximal-and-prime-ideals noetherian integral-domain
$endgroup$
add a comment |
$begingroup$
In this question I asked: "Are unique prime ideal factorization domains noetherian?".
In this answer Badam Baplan pointed out that locally noetherian domains are unique prime ideal factorization domains (UPIFD), that certain non-noetherian domains are locally noetherian (the first example seems to have been given by N. Nakano in 1953), and thus that these non-noetherian domains are UPIFD.
This prompts the present question:
Are unique prime ideal factorization domains locally noetherian?
Edit. For the sake of completeness recall that a UPIFD is a domain satisfying the following condition:
If $mathfrak p_1,dots,mathfrak p_k$ are distinct nonzero prime ideals of $A$, and if $m$ and $n$ are distinct elements of $mathbb N^k$, then we have
$$
mathfrak p_1^{m_1}cdotsmathfrak p_k^{m_k}nemathfrak p_1^{n_1}cdotsmathfrak p_k^{n_k}.
$$
commutative-algebra maximal-and-prime-ideals noetherian integral-domain
$endgroup$
$begingroup$
In my opinion this property is not enough to imply noetherianity, not even locally.
$endgroup$
– user26857
Jan 5 at 19:32
$begingroup$
Crossposted on MathOverflow: mathoverflow.net/q/320787/461
$endgroup$
– Pierre-Yves Gaillard
Jan 13 at 12:48
$begingroup$
The question has been answered on MathOverflow by Dario Spirito: mathoverflow.net/a/321090/461
$endgroup$
– Pierre-Yves Gaillard
Jan 17 at 11:54
add a comment |
$begingroup$
In this question I asked: "Are unique prime ideal factorization domains noetherian?".
In this answer Badam Baplan pointed out that locally noetherian domains are unique prime ideal factorization domains (UPIFD), that certain non-noetherian domains are locally noetherian (the first example seems to have been given by N. Nakano in 1953), and thus that these non-noetherian domains are UPIFD.
This prompts the present question:
Are unique prime ideal factorization domains locally noetherian?
Edit. For the sake of completeness recall that a UPIFD is a domain satisfying the following condition:
If $mathfrak p_1,dots,mathfrak p_k$ are distinct nonzero prime ideals of $A$, and if $m$ and $n$ are distinct elements of $mathbb N^k$, then we have
$$
mathfrak p_1^{m_1}cdotsmathfrak p_k^{m_k}nemathfrak p_1^{n_1}cdotsmathfrak p_k^{n_k}.
$$
commutative-algebra maximal-and-prime-ideals noetherian integral-domain
$endgroup$
In this question I asked: "Are unique prime ideal factorization domains noetherian?".
In this answer Badam Baplan pointed out that locally noetherian domains are unique prime ideal factorization domains (UPIFD), that certain non-noetherian domains are locally noetherian (the first example seems to have been given by N. Nakano in 1953), and thus that these non-noetherian domains are UPIFD.
This prompts the present question:
Are unique prime ideal factorization domains locally noetherian?
Edit. For the sake of completeness recall that a UPIFD is a domain satisfying the following condition:
If $mathfrak p_1,dots,mathfrak p_k$ are distinct nonzero prime ideals of $A$, and if $m$ and $n$ are distinct elements of $mathbb N^k$, then we have
$$
mathfrak p_1^{m_1}cdotsmathfrak p_k^{m_k}nemathfrak p_1^{n_1}cdotsmathfrak p_k^{n_k}.
$$
commutative-algebra maximal-and-prime-ideals noetherian integral-domain
commutative-algebra maximal-and-prime-ideals noetherian integral-domain
edited Jan 5 at 12:33
Pierre-Yves Gaillard
asked Jan 5 at 11:47
Pierre-Yves GaillardPierre-Yves Gaillard
13.4k23184
13.4k23184
$begingroup$
In my opinion this property is not enough to imply noetherianity, not even locally.
$endgroup$
– user26857
Jan 5 at 19:32
$begingroup$
Crossposted on MathOverflow: mathoverflow.net/q/320787/461
$endgroup$
– Pierre-Yves Gaillard
Jan 13 at 12:48
$begingroup$
The question has been answered on MathOverflow by Dario Spirito: mathoverflow.net/a/321090/461
$endgroup$
– Pierre-Yves Gaillard
Jan 17 at 11:54
add a comment |
$begingroup$
In my opinion this property is not enough to imply noetherianity, not even locally.
$endgroup$
– user26857
Jan 5 at 19:32
$begingroup$
Crossposted on MathOverflow: mathoverflow.net/q/320787/461
$endgroup$
– Pierre-Yves Gaillard
Jan 13 at 12:48
$begingroup$
The question has been answered on MathOverflow by Dario Spirito: mathoverflow.net/a/321090/461
$endgroup$
– Pierre-Yves Gaillard
Jan 17 at 11:54
$begingroup$
In my opinion this property is not enough to imply noetherianity, not even locally.
$endgroup$
– user26857
Jan 5 at 19:32
$begingroup$
In my opinion this property is not enough to imply noetherianity, not even locally.
$endgroup$
– user26857
Jan 5 at 19:32
$begingroup$
Crossposted on MathOverflow: mathoverflow.net/q/320787/461
$endgroup$
– Pierre-Yves Gaillard
Jan 13 at 12:48
$begingroup$
Crossposted on MathOverflow: mathoverflow.net/q/320787/461
$endgroup$
– Pierre-Yves Gaillard
Jan 13 at 12:48
$begingroup$
The question has been answered on MathOverflow by Dario Spirito: mathoverflow.net/a/321090/461
$endgroup$
– Pierre-Yves Gaillard
Jan 17 at 11:54
$begingroup$
The question has been answered on MathOverflow by Dario Spirito: mathoverflow.net/a/321090/461
$endgroup$
– Pierre-Yves Gaillard
Jan 17 at 11:54
add a comment |
0
active
oldest
votes
Your Answer
StackExchange.ifUsing("editor", function () {
return StackExchange.using("mathjaxEditing", function () {
StackExchange.MarkdownEditor.creationCallbacks.add(function (editor, postfix) {
StackExchange.mathjaxEditing.prepareWmdForMathJax(editor, postfix, [["$", "$"], ["\\(","\\)"]]);
});
});
}, "mathjax-editing");
StackExchange.ready(function() {
var channelOptions = {
tags: "".split(" "),
id: "69"
};
initTagRenderer("".split(" "), "".split(" "), channelOptions);
StackExchange.using("externalEditor", function() {
// Have to fire editor after snippets, if snippets enabled
if (StackExchange.settings.snippets.snippetsEnabled) {
StackExchange.using("snippets", function() {
createEditor();
});
}
else {
createEditor();
}
});
function createEditor() {
StackExchange.prepareEditor({
heartbeatType: 'answer',
autoActivateHeartbeat: false,
convertImagesToLinks: true,
noModals: true,
showLowRepImageUploadWarning: true,
reputationToPostImages: 10,
bindNavPrevention: true,
postfix: "",
imageUploader: {
brandingHtml: "Powered by u003ca class="icon-imgur-white" href="https://imgur.com/"u003eu003c/au003e",
contentPolicyHtml: "User contributions licensed under u003ca href="https://creativecommons.org/licenses/by-sa/3.0/"u003ecc by-sa 3.0 with attribution requiredu003c/au003e u003ca href="https://stackoverflow.com/legal/content-policy"u003e(content policy)u003c/au003e",
allowUrls: true
},
noCode: true, onDemand: true,
discardSelector: ".discard-answer"
,immediatelyShowMarkdownHelp:true
});
}
});
Sign up or log in
StackExchange.ready(function () {
StackExchange.helpers.onClickDraftSave('#login-link');
});
Sign up using Google
Sign up using Facebook
Sign up using Email and Password
Post as a guest
Required, but never shown
StackExchange.ready(
function () {
StackExchange.openid.initPostLogin('.new-post-login', 'https%3a%2f%2fmath.stackexchange.com%2fquestions%2f3062641%2fare-unique-prime-ideal-factorization-domains-locally-noetherian%23new-answer', 'question_page');
}
);
Post as a guest
Required, but never shown
0
active
oldest
votes
0
active
oldest
votes
active
oldest
votes
active
oldest
votes
Thanks for contributing an answer to Mathematics Stack Exchange!
- Please be sure to answer the question. Provide details and share your research!
But avoid …
- Asking for help, clarification, or responding to other answers.
- Making statements based on opinion; back them up with references or personal experience.
Use MathJax to format equations. MathJax reference.
To learn more, see our tips on writing great answers.
Sign up or log in
StackExchange.ready(function () {
StackExchange.helpers.onClickDraftSave('#login-link');
});
Sign up using Google
Sign up using Facebook
Sign up using Email and Password
Post as a guest
Required, but never shown
StackExchange.ready(
function () {
StackExchange.openid.initPostLogin('.new-post-login', 'https%3a%2f%2fmath.stackexchange.com%2fquestions%2f3062641%2fare-unique-prime-ideal-factorization-domains-locally-noetherian%23new-answer', 'question_page');
}
);
Post as a guest
Required, but never shown
Sign up or log in
StackExchange.ready(function () {
StackExchange.helpers.onClickDraftSave('#login-link');
});
Sign up using Google
Sign up using Facebook
Sign up using Email and Password
Post as a guest
Required, but never shown
Sign up or log in
StackExchange.ready(function () {
StackExchange.helpers.onClickDraftSave('#login-link');
});
Sign up using Google
Sign up using Facebook
Sign up using Email and Password
Post as a guest
Required, but never shown
Sign up or log in
StackExchange.ready(function () {
StackExchange.helpers.onClickDraftSave('#login-link');
});
Sign up using Google
Sign up using Facebook
Sign up using Email and Password
Sign up using Google
Sign up using Facebook
Sign up using Email and Password
Post as a guest
Required, but never shown
Required, but never shown
Required, but never shown
Required, but never shown
Required, but never shown
Required, but never shown
Required, but never shown
Required, but never shown
Required, but never shown
$begingroup$
In my opinion this property is not enough to imply noetherianity, not even locally.
$endgroup$
– user26857
Jan 5 at 19:32
$begingroup$
Crossposted on MathOverflow: mathoverflow.net/q/320787/461
$endgroup$
– Pierre-Yves Gaillard
Jan 13 at 12:48
$begingroup$
The question has been answered on MathOverflow by Dario Spirito: mathoverflow.net/a/321090/461
$endgroup$
– Pierre-Yves Gaillard
Jan 17 at 11:54